-4x + 3y = -2

Y = x - 1

Answers

Answer 1

Answer:

(-1, -2)

General Formulas and Concepts:

Pre-Algebra

Order of Operations: BPEMDAS

Brackets Parenthesis Exponents Multiplication Division Addition Subtraction Left to Right  

Distributive Property

Equality Properties

Multiplication Property of Equality Division Property of Equality Addition Property of Equality Subtraction Property of Equality

Algebra I

Terms/CoefficientsSolving systems of equations using substitution/elimination

Step-by-step explanation:

Step 1: Define

-4x + 3y = -2

y = x - 1

Step 2: Solve for x

Substitution

Substitute in y:                                                                                                  -4x + 3(x - 1) = -2[Distributive Property] Distribute 3:                                                                 -4x + 3x - 3 = -2Combine like terms:                                                                                         -x - 3 = -2[Addition Property of Equality] Add 3 on both sides:                                     -x = 1[Division Property of Equality] Divide -1 on both sides:                                 x = -1

Step 3: Solve for y

Substitute in x [Original Equation]:                                                                  y = -1 - 1Subtract:                                                                                                            y = -2

Related Questions

someone please help me answer this!!

Answers

Answer:

100 in is the answer.

Step-by-step explanation:

a = 60 in

b = 80 in

c = ?

According to the Pythagorean theorem,

a² + b² = c²

60² + 80² = c²

3600 + 6400 = c²

c² = 10000

c = 100 in

∴ the distance between the opposite corners of the bed is 100 in

Rachel downloaded a movie at a constant speed.
After she downloaded 40% of the movie, the remaining download time was 40 seconds.
After she downloaded 40 mb, the remaining download time was 16 seconds.
Compute the total size of the movie (in mb) rounded to the nearest mb.

Answers

Answer:

The total weight of the file was 64 mb.

Step-by-step explanation:

Given that Rachel downloaded a movie at a constant speed, and after she downloaded 40% of the movie, the remaining download time was 40 seconds, while after she downloaded 40 mb, the remaining download time was 16 seconds, to determine the total size of the movie rounded to the nearest mb, the following calculation must be performed:

100 - 40 = 60

60% = 40 seconds

40 mb = 16 seconds left

16/40 = 4/10

60/40 = 1.5

1.5% for every 1 second of download.

16 x 1.5 = 24

40 + 24 = 64

Thus, the total weight of the file was 64 mb.

answer this please!!!

Answers

Answer:

yes. It's a right triangle.

Step-by-step explanation:

According to the Pythagoras theorem,

a² + b² = c²

20² + 21² = 29²

400 + 441 = 841

841 = 841

LHS = RHS

Which following relations represent a function

Answers

Answer:

I think that actually all of them represent a function , except D .

hope it was helpful .

Write an equation for a circle that follows the same pattern as the other circles. Explain how your equation follows the same pattern you identified.

Answers

9514 1404 393

Answer:

  (x -4)² +(y +3)³ = 4

Step-by-step explanation:

Each circle is centered on (h, k) = (4, -3). Each circle has a radius 1 unit smaller than the circle just outside of it. The innermost circle shown has a radius of 3, so the next in the pattern of smaller circles will have a radius of 2. The equation for a circle centered at (h, k) with radius r is ...

  (x -h)² +(y -k)² = r²

The next circle in the pattern is centered at (4, -3) and has radius 2, so its equation is ...

  (x -4)² +(y +3)² = 4

Please Help Me Fast.
FAST FAST FAST

Answers

Answer:

Easy it's 40

Step-by-step explanation:

I think it's 40 because the other numbers are all with in the range of 10-20 except 40.

Answer:

Answer is 40! :)

Answer to this problem

Answers

The 3 deals are worth the same price per unit of $0.1 a pencil.

Price per unit of items

Based on the given prices, we can calculate the price per pencil for each pack:

Pack of 12 pencils: $1.29

       Price per pencil = $1.29 / 12 = $0.1075 (approximately $0.11)

Pack of 24 pencils: $2.35

       Price per pencil = $2.35 / 24 = $0.0979 (approximately $0.10)

Pack of 50 pencils: $5.45

       Price per pencil = $5.45 / 50 = $0.109 (approximately $0.11)

In other words, the 3 deals are worth the same price per unit of the pencil.

More on  price per unit problems can be found here: https://brainly.com/question/13646105

#SPJ1

A classroom had 16 computers. If the ratio of computers to ipads was 4 : 5, how many ipads does the classroom have?

Answers

Answer:

20 ipads

Step-by-step explanation:

The ratio is 4 : 5

      4:5

*4             *4

   16c :20i

16 Computers and 20 Ipads.

20 iPads

4 computers : 5 iPads is the ratio

16 computers means we’re multiplying the ratio x 4. 5x4 = 20

You can also set it up to cross-multiply and divide:

4/5 = 16/x
4x = 80
x = 20

simplify the following complex number, without changing the polar form and leave your answer in polar form:
[tex]\frac{(2\ \textless \ \frac{2\pi }{3})^{3} (3\ \textless \ \frac{3\pi }{2}) }{4\ \textless \ \frac{7\pi }{3} }[/tex]

Answers

The simplified form of the complex number in polar form is: 2 < (-7π/6)

How to simplify the complex number

To simplify the given complex number in polar form, we can apply the rules of exponentiation and division for complex numbers:

First, let's simplify the numerator:

(2 < 2π/3)^3 = (2^3) < (2π/3 * 3) = 8 < (2π)

(3 < 3π/2) remains the same.

Now, let's simplify the denominator:

4 < (7π/3) remains the same.

Next, let's divide the numerator by the denominator:

[(8 < (2π)) * (3 < (3π/2))] / (4 < (7π/3))

Using the rules of division for complex numbers, we can divide the magnitudes and subtract the angles:

(8/4) < (2π - 7π/3 - (3π/2)) = 2 < (2π/3 - 3π/2 - π/6) = 2 < (-7π/6)

Therefore, the simplified form of the complex number in polar form is:

2 < (-7π/6)

Learn more about complex numbers at https://brainly.com/question/29747482

#SPJ1

HELP! How many triangles can be made from the following three lengths: 1.6 centimeters, 7.4 centimeters, and 5.9 centimeters?

none

one

more than one

Answers

Answer: one

Step-by-step explanation:

Theo is using the Quadratic Formula to solve a quadratic equation. Which of the following is the next step for simplifying

x=−4±4√6/2

Answers

Answer: look at the picture

Step-by-step explanation:

Removing the denominator by dividing both the numerator's terms by 2 is the needed next step for simplification. The result is: x=−2±2√6

How to find the solution to a standard quadratic equation?

Suppose the given quadratic equation is

[tex]ax^2 + bx + c = 0[/tex]

Then its solutions are given as

[tex]x = \dfrac{-b \pm \sqrt{b^2 - 4ac}}{2a}[/tex]

Here, we're given the expression:

[tex]x = \dfrac{-4 \pm 4\sqrt{6}}{2}[/tex]

The next step to simplify is to make it look more simple.

[tex]x = \dfrac{-4}{2} \pm \dfrac{4\sqrt{6}}{2}\\\\x = -2 \pm 2\sqrt{6}[/tex]

Thus, removing the denominator by dividing both the numerator's terms by 2 is the needed next step for simplification. The result is: x=−2±2√6

Learn more about quadratic equations here:

https://brainly.com/question/3358603

Find the volume of a cone with a base diameter of 10 cm and a height of 9 cm

Answers

Answer:

235.71cm^2

Step-by-step explanation:

Volume of cone = 1/3 pie r^2 h

1/3*22/7*5*5*9

= 235.71cm^2

Volume of a cone is 235.6 cm³.

Why is the volume of a cone?The volume of a cone with height h and radius r is 1/3πr2h, which is exactly one third the volume of the smallest cylinder that it fits inside. This can be proved easily by considering a cone as a solid of revolution, but I would like to know if it can be proved or at least visual demonstrated without using calculus.  

The volume of a cone is calculated the using the formula:

[tex]V = \frac{1}{3} \pi r^{3} h[/tex]

Given,

         height of the cylinder is, h=9cm.

         The diameter of the base is 10cm. r = 5 cm

    put the values into the formula to get -

          V = [tex]\frac{1}{3} * \pi * 5^{2} * 9[/tex]

         [tex]V = 75 \pi[/tex]

Put value of π = 3. 14 ,

          V = 75 * 3.14

           V =  235.6 cm³

Therefore, Volume of a cone is 235.6 cm³.

Learn more about Volume of a cone  brainly.com/question/1578538

#SPJ2

aced or higher please hellp i beg :C

Answers

Answer:

A: Find the area of the outer trapezoid and then subtract the area of the inner trapezoid

B: 15 square units

Step-by-step explanation:

A:

Find the area of the outer trapezoid and then subtract the area of the inner trapezoid

(because if the inner trapezoid wasn't there and you were looking for the shaded region, you'd find the area of the outer trapezoid. since you have to take away the area the inner trapezoid takes up, the shaded region = area of outer trapezoid - area of inner trapezoid)

B

The area of a trapezoid is ((a+b)/2)h, where a is the top or the trapezoid, b is the bottom and h is the height.

The top of the outer trapezoid ranges from 4 to 6, so a=6-4=2. The bottom of the outer trapezoid ranges from 2 to 8, so b=8-2=6. The height of the outer trapezoid ranges from 2 to 8, so h=8-2=6.

The area of the outer trapezoid = ((a+b)/2)h = ((2+6)/2)(6) = 4(6) = 24

The top of the inner trapezoid ranges from 4 to 6, so a=6-4=2. The bottom of the inner trapezoid ranges from 3 to 7, so b=7-3=4. The height of the inner trapezoid ranges from 3 to 6, so h=6-3=3.

The area of the inner trapezoid = ((a+b)/2)h = ((2+4)/2)(3) = 3(3) = 9

The are of the shaded region = the area of the outer trapezoid - the area of the inner trapezoid = 24 - 9 = 15 square units

In the figure, lines a and b are parallel. Which statements are true? Select each correct answer. Responses m∠2=m∠1 measure of angle 2 equals measure of angle 1 m∠3=m∠1 measure of angle 3 equals measure of angle 1 m∠1=75° measure of angle 1 equals 75 degrees m∠2=105° measure of angle 2 equals 105 degrees

Answers

The correct statements from the line are given as follows:

m∠2=m∠1m∠1=75°

What are corresponding angles?

When two parallel lines are cut by a transversal, corresponding angles are pairs of angles that are in the same position relative to the two parallel lines and also to the transversal.

Corresponding angles are congruent, which means that they have the same angle measure.

Two corresponding angles for this problem are given as follows:

<2 and <1.

Hence:

m < 2 = m < 1.

Angles 1 and 105º form a linear pair, meaning that they are supplementary, hence the measure of angle 1 is given as follows:

m < 1 + 105º = 180º.

m < 1 = 75º.

More can be learned about corresponding angles at brainly.com/question/24607467

#SPJ1

someone please help me answer this

Answers

Answer:

It would be 20

Answer:

6.4 in is the answer.

Step-by-step explanation:

a = 5 in

b = 4 in

c = ?

According to the Pythagorean theorem,

a² + b² = c²

5² + 4² = c²

25 + 16 = c²

c² = 41

c = 6.403

Round to the nearest tenth,

c = 6.4 in

∴ the length of the pencil is 6.4 in

QUICK HELP with the Image below

Answers

Answer:

12 8/11

Step-by-step explanation:

hope it helps you

What is 1/6 divided by 1/3 in simplest form

Answers

Answer:

1/2 or 0.5

Step-by-step explanation:

Answer:

1/18

Step-by-step explanation:

1/6÷1/3

1/3×1/6

1/3×6

1/18

What is the square root of –16?
A. –8i
B. –4i
C. 4i
D. 8i

Answers

Answer:

C. 4i

Step-by-step explanation:

Answer:

C. 4i

Step-by-step explanation:

[tex]\sqrt{-16} = \sqrt{16} \sqrt{-1}[/tex] (because [tex]\sqrt{ab} =\sqrt{a} \sqrt{b}[/tex])

Since [tex]\sqrt{16}[/tex]=4, and [tex]\sqrt{-1}[/tex]=i, [tex]\sqrt{-16}[/tex]=4i

The owner of a pet shop predicts that 10 out of 20 new puppies are male. He randomly selects 10 of the puppies, and notes that all 10 are male. Which of the following conclusions best supports his findings?
Select one:

10 of the puppies are male and 10 are female.

More than 10 of the puppies are male.

All 20 puppies are male.

Less than 10 of the puppies are male.

Answers

Answer:

A, 10 of the new puppies are male and 10 are female

Step-by-step explanation:

Really this is the one that makes the most since because if 10 of the puppies are male, then 10 must be female.

Answer: A

Step-by-step explanation:

Pictures in order !

Answers

Answer:X=10

Step-by-step explanation: 3x-2=2x+8 If that's what you're looking for

Will give Brainly

Find the probability that a
randomly
selected point within the circle falls
in the red shaded area.
60°
60°
r= 4 cm
[? ]%
Round to the nearest tenth of a percent.

Answers

Answer: 33.3%

s Step-by-step explanation: The combined arc length of the red area is 120, which is 1/3 of 360, the full arc. So the probability is 1/3, or 33.3%. Hope this helps!

The probability that a randomly selected point within the circle falls

in the red shaded area will be 33.33%.

What is probability?

Probability is defined as the ratio of the number of favorable outcomes to the total number of outcomes in other words the probability is the number that shows the happening of the event.

Probability = Number of favorable outcomes / Number of sample

Given that the shaded red sector has an angle of 60 degrees and the radius of the circle is 4 cm. The probability will be calculated as:-

The formula for a circle's sector's area is (Ф/360°) x π(r²), where r is the circle's radius and is the sector's angle.

Area of shaded region =  (Ф/360°) x π(r²),

Area of shaded region = ( 60 / 360 ) x π (4²)

Area of shaded region = 8.377 square cm

Total shaded area = 2 x 8. 377 = 16.75 square cm

Total area of circle = π (4²) = 50.265 square cm

The probability is calculated by the formula below:-

Probability = Number of favorable outcomes / Number of sample

Number of favorable outcomes = 8.377

Number of sample = 50.265 square cm

Probability = 8.377 / 50.265 = 0.3333 = 33.33%

Therefore, the probability that a randomly selected point within the circle falls in the red-shaded area will be 33.33%.

To know more about probability follow

https://brainly.com/question/27930712

#SPJ5

Find the value of x.


help please !

Answers

Answer:

33

Step-by-step explanation:

The sum of the three interior angles in a triangle is ALWAYS equivalent to 180 degrees.

69 + 78 = 147

180 - 147 = 33

Hope this is helpful & accurate! Best wishes.

Find the equation of the line specified.
The slope is -7, and it passes through ( 5, -3).
a.
y = -7x - 3
c.
y = -14x + 32
b.
y = -7x + 32
d.
y = -7x - 38

Answers

Answer and working out attached below. Hope it helps

Which graph represents exponential decay?

On a coordinate plane, a straight line has a negative slope.

On a coordinate plane, a curve approaches y = 0 in quadrant 2 and curves up in quadrant 1.

On a coordinate plane, a straight line has a positive slope.

On a coordinate plane, a curve approaches y = 0 in quadrant 1 and curves up in quadrant 2.

Answers

The graph which shows the function is exponential decay is on a coordinate plane, a curve approaches y = 0 in quadrant 1 and curves up in quadrant 2 thus, option (D) is correct.

What is an exponential function?

In mathematics, an exponential function is a relationship of the type y = ax, where x is an independent variable that spans the entire real number line and is expressed as the exponent of a positive number.

As per the given,

The exponential function is a function that changes rapidly, for example, y = 6^x here as x approaches higher the y approaches rapidly higher.

Exponential decay is the exponential function that rapidly decreases as x increases for example y = 6^(-x).

The graph of the exponential decay has been graphed below.

Hence "The graph which shows the function is exponential decay is on a coordinate plane, a curve approaches y = 0 in quadrant 1 and curves up in quadrant 2".

For more about exponential function,

https://brainly.com/question/15352175

#SPJ6

Answer:

D

Step-by-step explanation:

edge

hope this helps :)

Find the area of the figure.

Answers

Answer:

91

Step-by-step explanation:

Answer:

91

Step-by-step explanation:

(11 + 15) × 7 ÷ 2 = 26 × 7 ÷ 2 = 91

Denise made 1/2 of a liter of hot chocolate. Each mug holds 1/10 of a liter. How many mugs will Denise be able to fill?

Answers

Answer:

5 mugs

Step-by-step explanation:

1/2 converted into 10ths would be 5/10 ... meaning 5 mugs

pls help
#1
isjsbijsj​

Answers

Answer:

The value of “Y” is (6)

Step-by-step explanation:

the x value unit of “5” has the y value of “6.”

Answer:  6

EXPLANATION:

The x axis is the one on the bottom that goes like this : ---

The y axis is the one that is like a vertical line up like this: |

If you look at the x axis you can see that one the 5 and you go up you can see there is a dot that is on the 6.

Solve the Following inequality by choosing all of Possible range of Values that make the Statement True.

2 (x) + 12 ≤ 30

Group of answer choices

9, 10, 11

7, 8, 9

-7, -8, -9

9.001, 9.002, 9.003

Answers

Answer:

7, 8, 9

-7, -8, -9

Step-by-step explanation:

2(x) + 12 ≤ 30

2x + 12 ≤ 30

2x ≤ 30 - 12

2x ≤ 18

x ≤ 9

x has to be less than OR equal to 9

find the value of x

HELP please !

Answers

Answer:

116+34+x=180

150+x=180

x=180-150

x=30

what % of 4 is 7

i need this to be answered as quickly as possible please

Answers

Answer:

the answer

= 17.5

Step-by-step explanation:

https://www.percentagecal.com/answer/.7-is-what-percent-of-4#:~:text=Percentage%20Calculator%3A%20.,%3D%2017.5

Other Questions
Select a specific institution. Identify this institution and utilizing its published MCR analyze costs and revenue of at least 3 specific departments. Then, evaluate implications of this volume to the facilitys future income stream. Question 3 Question 3a Assume you are the Minister of Finance and Economic Planning for Ghana, in charge of Fiscal Policy. The Research Director of the Ministry brought you the following data on Ghana for the previous fiscal year, 2021. An examination of the data reveals that, during the fiscal year 2021, households in Ghana saved 20% of their disposable income (Y) and spent the rest on consumption. In addition, GH5,000.00 was spent on Consumption expenditure (C), which is independent of income and Gross Private Investment (I) was GH7,000.00. Total Government expenditure (G) which stood at GHe8,000.00 was supposed to be financed by a lump sum tax of GH2,000.00 (independent of income) and a proportional tax rate of 25% of national income. Exports (X) stood at GH 2,500.00. In addition, the country's import (M) during the previous fiscal year comprises of GH1,000.00 which was independent of the country's national income and 10% which was dependent of the country's national income. Given these data on Ghana for the previous year: i. Compute the equilibrium level of income (Y), Consumption (C), Tax (T) and Savings (S). (Hint: C = a + byd; T = To+tY and M = M + mY) Determine the Government fiscal stance. (10 marks) (1 mark) iii. If the full employment level of national income is GH40,000.00, determine the income gap. iv. What fiscal policy would be appropriate to address this gap? (1 mark) (1 mark) V. If there is an increase in export to GH4,000.00, find the new level of equilibrium income. (2 marks) vi. Show how a GH 2,000 increase in government spending financed by a GH$2,000 increase in taxes will affect the level of national income. (2 marks) Question 3b Gross Domestic Product (GDP) is not a good measure of welfare in an economy. Discuss. (3 marks) A supermarket CEO claims that 26% of customers that enter the store purchase milk or bread. A survey of 320 customer showed that 67 customers purchased bread or milk on their trip to the store. Assuming the CEO's claim is correct, determine (to 4 decimal places):1. the standard error for the sampling distribution of the proportion.2. the probability that the sample proportion is no more than that found in the survey. Have China, India, Japan, South Korea and Singapore experiencedthe catch-up effect (also known as convergence theory)? Explaindifferences with Australia and the USA. Discount loan (interest and principal at maturity). Chuck Ponzi has talked an elderly woman into loaning him $10,000 for a new business venture. She has, however, successfully passed a finance class and requires Chuck to sign a binding contract on repayment of the $10,000 with an annual interest rate of 11% over the next 15 years. Determine the cash flow to the woman under a discount loan, in which Ponzi will have a lump-sum payment at the end of the contract. What is the amount of payment that the woman will receive at the end of years 1 through 14? A company produces packets of soap powder labeled "Giant Size 32 Ounces. The actual weight of soap powder in such a box has a Normal distribution, with a mean of 33 or and a standard deviation of 0.3 or. To ad having dissatisfied customers, the company says a box of soap is considered underweight if it weighs less than 32 or. To avoid losing money, it labels the top 5% (the heaviest 5%) overweight. How heavy does a bes have to be for it to be labeled overweight? 33.82 or 634,15 or 34.6202 31.80 o QUESTION 7 Some researchers have noted that adolescents who spend a lot of time playing video or computer games are at greater risk for depression and violence. This is an example of a a single-blind experiment, because the subjects knew they were playing games ha valid conclusion, because more time yields more aggression is a positive association c. a paired data experiment, because we are studying both aggression and game playing dan observational study with larking variables that may explain the association. QUESTIONS A company produces packets of soap poeder held"Chante 12 mes. The actual weight of top powder in a box una distribution of 33 oranda de roso having dised customers, the company box fit considered underwright if it weh less 1902. To avoiding help the best way does best be for is to be weled overweight XXX b.34.15 34.620 0.31.80 OR QUESTION 4 The time needed for college students to complete a certainpaper and pencil maze follows a Normal distribution, with a mean of 80 seconds and a standard deviation of 6 seconds. You wish to see if the mean time je is changed by meditation, so you have a group of 8 college students meditate for 30 minutes and then complete the mare. It takes them an average of 1-74 seconds to complete the mare. Use this information to test the hypotheses Hg: -80, Ha80 at significance level a-0.02. You conclude that a. Ha should be accepted. bHo should be rejected. Hy should not be rejected. d. this is a borderline case and so decision should be made. 11 12 15 16 17 22 24 29 33 38 what is the median An account linked with another account that has an opposite normal balance and that is subtracted from the balance of the related account is a(n):A) Accrued account.B) Contra account.C) Temporary Account.D) Clearing account.E) Permanent Account. Explain the Decision-making method (High and low Methods) withexamples. Find the non-parametric equation of the plane with normal (5,6,6)-5,6,6 which passes through point (5,6,0)5,-6,0.Write your answer in the form Ax+By+Cz+d=0Ax+By+Cz+d=0 using lower case x,y,zx,y,z and * for multiplication. Please Do Not rescale (simplify) the equation. Let X1, X2, ..., Xm be a random sample from a population with mean mu1 and variance of sigma1^2=, and let Y1, Y2, ... , Yn be a random sample from a population with mean mu2 and variance sigma2^2, and that X and Y samples are independent of one another. Which of the following statements are true? Xbar is normally distributed with expected value mu1 and variance sigma1^2/m Ybar is normally distributed with expected value mu2 and variance sigma2^2/m Xbar-Ybar is normally distributed with expected value mul-mu2 and variance (sigma1^2/m+sigma2^2/n). Xbar-Ybar is an unbiased estimator of mul-mu2. All of the above statements are true. Touch may communicate many different meanings includinga.all of the above. b.control. c.positive emotionsd.playfulness. Explain the full story of Garland, including up to Amy Coney Barrett. Did Trump and Mitch McConnell as in accordance with the constitution? As CRO of Lakeside Bank, a bank that has 15 million of fixed-rate assets, 30 million of rate-sensitive assets, 25 million of fixed-rate liabilities, and 20 million of rate-sensitive liabilities. Conduct a gap analysis for Lakeside Bank and show what will happen to bank profits if interest rates rise by 4 percentage points. Answer all parts (a) to (e) of this question. If a firm produces quantities q1 and q2 of two goods, its total cost is: C = q1 + q 2 1 + q 2 2 q1q2. The goods are sold in competitive markets at prices p1 > 1 and p2.(a) [10 marks] Write down an expression for the profit of the firm. Obtain and provide an economic interpretation for the first-order profit-maximising conditions.(b) [10 marks] Using the Cramers rule, find the quantities q1 and q2 that satisfy the first order conditions.(c) [10 marks] Find the second-order conditions for profit maximisation. For what values of are the second-order conditions satisfied?(d) [10 marks] Assume the second-order condition is satisfied. Use calculus to determine the way in which the supply of good 2 varies with a rise in p1. Explain why it depends on the sign of .(e) [10 marks] Assume the second-order condition is satisfied. If p1 = 0.5 and p2 = 1, under which condition about will good 1 be supplied by the firm? Explain the economic intuition behind the results. Cullumber Corporation issued 368 shares of $10 par value ordinary shares and 123 shares of $50 par value preference shares for a lump sum of $16,587. The ordinary shares have a market price of $20 per share, and the preference shares have a market price of $90 per share. Prepare the journal entry to record the issuance. Consider the set S = {(o,p,q,r): op-qr =0 }Provide a counterexample to show that this set is not a subspace of R4 If the alternate hypothesis is justifiably directional (rather than non-directional), what should the researcher do when conducting a t test? O a one-tailed test O a two-tailed test O set the power to equal B O set to be less than the significance level a 0.20 m solution contains 6.4 g of so2. what is the volume of the solution? report your answer with two significant figures. Fulton is employed at an annual salary of S22,532 paid semi monthly. The regular workwerk in 36 hours (a) What is the regular salary per pay period? (b) What is the hourly rate of pay? c) What is the gross pay for a pay period in which the employee worked 9 hours overtime at time and one half regular pay?